How to prove an inequality for a definite exponential integral

In summary, the conversation was about trying to prove an inequality using simulations and splitting the range of integration. An expert provided a summary of the conversation and presented a proof using integrals and a change of variables. This proof was considered to be elegant and effective by the participants.
  • #1
paola
2
0
Hello gurus,

I've been trying to prove the following inequality for several days:

[itex]\int_1^\infty \frac{\exp\left(-\frac{(x-1)^2}{2a^2}\right)}{x}dx > \ln(1+a)\quad \forall a>0. [/itex]

I've demonstrated by simulations that this inequality holds. I‘ve also proved that this inequality holds for large [itex]a[/itex]. But, proving [itex]\forall a[/itex] exhausted me...

Who can help? Many thanks!

Paola
 
Last edited:
Physics news on Phys.org
  • #2
paola said:
I've been trying to prove the following inequality for several days:

[itex]\int_1^\infty \frac{\exp\left(-\frac{(x-1)^2}{2a^2}\right)}{x}dx > \ln(1+a)\quad \forall a>0. [/itex]

I've demonstrated by simulations that this inequality holds. I‘ve also proved that this inequality holds for large [itex]a[/itex]. But, proving [itex]\forall a[/itex] exhausted me...
A possibility is to split the range of integration at 1+a. The 1 to 1+a part then looks to be a bit less than ln(1+a)
[itex]\int_1^{a+1} \frac{\exp\left(-\frac{(x-1)^2}{2a^2}\right)}{x}dx = \int_0^a \frac{\exp\left(-\frac{x^2}{2a^2}\right)}{1+x}dx [/itex]
We can then try to top it up to reach ln(1+a) by folding back the rest of the range, e.g. x = (a+1)2/y:
[itex]\int_{a+1}^\infty \frac{\exp\left(-\frac{(x-1)^2}{2a^2}\right)}{x}dx = [/itex][itex]-\int_{a+1}^0 \frac{\exp\left(-\frac{((a+1)^{2}/y-1)^2}{2a^2}\right)}{y}dy [/itex]
So if you could show that
[itex]\exp\left(-\frac{x^2}{2a^2}\right) + \exp\left(-\frac{((a+1)^{2}/(x+1)-1)^2}{2a^2}\right)[/itex] >= 1
your result would follow.
But note that in the folding back the integration range became x = -1 to a. So we've wasted some in the -1 to 0 range. That might be critical. If so, a possible way to fix that is to break it further into ranges a to a2, to a3 and so on, folding each past a back onto the target range.
 
  • #3
haruspex said:
So if you could show that
[itex]\exp\left(-\frac{x^2}{2a^2}\right) + \exp\left(-\frac{((a+1)^{2}/(x+1)-1)^2}{2a^2}\right)[/itex] >= 1
.. which seems to be true for a < 2, but it breaks much above that. Asymptotically, looks like it goes as low as 0.95 when x = 0.34*a.
 
  • #4
Let f(a) be the integral on the left. We can write:

$$ f(a) = \int_0^\infty \frac{1}{x+1} \exp(\frac{-x^2}{2a^2})\, dx.$$

Let g(a)= ln(1+a). f(0) = g(0) = 0. For f you can sub y = ax and take the limit as a goes to zero.

[itex] \frac{1}{a\sqrt{\pi/2}}\int_0^\infty h(x) \exp(\frac{-x^2}{2a^2})\,dx [/itex] goes to h(0) as a goes down to zero. This shows that [itex] f'(0) = \sqrt{\pi/2} > g'(0)[/itex].

This proves the inequality at least in a neighborhood of 0.

[EDIT] fixed confusing notation that used g(x) in two different senses in the same line.
 
Last edited:
  • #5
Cool problem. I think I have it now, but I do not guarantee the nonexistence of errors or vastly simpler proofs...

Notation:

[itex] g(x) = \frac{1}{1+x} [/itex]

Notice that [itex] \log(a+1) = \int_0^a g(x)\, dx = \int_0^\infty g(x) \chi_{[0,a]}(x)\, dx [/itex].

So we want to prove:
$$ \int_0^\infty g(x) e^{-x^2/2a^2}\, dx > \int_0^\infty g(x) \chi_{[0,a]}(x)\, dx.$$

Subtract from both sides to reformulate this inequality as [itex] \int_0^\infty g(x)H(x/a)\, dx > 0 [/itex]. By a change of variables, we get
$$ \int_0^\infty ag(ax) H(x)\, dx >0 ,$$
where [itex] H(x) = e^{-x^2/2} - \chi_{[0,1]}(x) [/itex]. By setting b= 1/a, we get:
$$ \int_0^\infty \frac{1}{x+b} H(x)\, dx >0 .$$

I will just point out that if b=0, this integral is well defined and positive as can be verified through Wolfram alpha or some analytic technique. We break up the integral into two parts and wish to prove that for any positive b,
$$ \int_0^1 \frac{1}{x+b} (1-e^{-x^2/2})\, dx < \int_1^\infty \frac{1}{x+b}e^{-x^2/2}\, dx.$$

The left integrand is less than [itex] \frac{1}{x}\frac{1}{1+b} (1-e^{-x^2/2}).[/itex]
The right integrand is greater than [itex] \frac{1}{x}\frac{1}{1+b} e^{-x^2/2}.[/itex] Therefore the inequality is true if
$$ \frac{1}{1+b} \int_0^1 \frac{1}{x}(1-e^{-x^2/2})\, dx < \frac{1}{1+b}\int_1^\infty \frac{1}{x}e^{-x^2/2}\, dx.$$

So it is reduced to a single integral inequality which can be checked explicitly. In fact, this is the same as the inequality above that said was true by Wolfram Alpha.
 
  • #6
Very neat, Vargo. Looks right to me.
 
  • #7
haruspex and Vargo, thank you so much!

Vargo, how beautiful your proof is! Thanks a lot!
 

1. Can I use the same techniques to prove an inequality for a definite exponential integral as I would for a regular integral?

Yes, many of the same techniques used to prove inequalities for regular integrals can also be applied to definite exponential integrals. However, there may be some additional steps or considerations that need to be taken into account due to the presence of the exponential function.

2. What are some common techniques used to prove inequalities for definite exponential integrals?

Some common techniques include using properties of the exponential function, manipulating the integral to make it easier to evaluate, and using mathematical induction for more complicated inequalities.

3. Are there any specific properties of the exponential function that are helpful in proving inequalities for definite exponential integrals?

Yes, some helpful properties include the fact that the exponential function is always positive, it is strictly increasing, and it has a unique inverse function. These properties can be used to simplify the integral or manipulate it into a more manageable form.

4. How can I determine the bounds for a definite exponential integral in order to prove an inequality?

The bounds for a definite exponential integral can be determined by considering the behavior of the function being integrated and the values of the exponential function at those bounds. It may also be helpful to sketch a graph of the function to visualize the bounds.

5. Are there any special cases or exceptions to consider when proving inequalities for definite exponential integrals?

Yes, there are certain cases where the usual techniques may not apply, such as when the upper bound of the integral is infinity or when the function being integrated is not continuous. In these cases, alternative approaches may be needed to prove the inequality.

Similar threads

Replies
4
Views
989
Replies
21
Views
818
Replies
5
Views
385
Replies
3
Views
970
  • Topology and Analysis
Replies
4
Views
275
Replies
2
Views
942
  • Calculus
Replies
6
Views
1K
Replies
3
Views
1K
Back
Top